1answer.
Ask question
Login Signup
Ask question
All categories
  • English
  • Mathematics
  • Social Studies
  • Business
  • History
  • Health
  • Geography
  • Biology
  • Physics
  • Chemistry
  • Computers and Technology
  • Arts
  • World Languages
  • Spanish
  • French
  • German
  • Advanced Placement (AP)
  • SAT
  • Medicine
  • Law
  • Engineering
olchik [2.2K]
2 years ago
10

Please help asap!!!!!​

Mathematics
1 answer:
Harrizon [31]2 years ago
8 0

Answer:

-7

Step-by-step explanation:

Solve:

  • \sqrt[3]{-343}
  • \sqrt[3]{-1(7 * 7*7)}
  • \sqrt[3]{-1} * \sqrt[3]{7*7*7}
  • -1 * 7
  • -7

The answer is \boxed{-7}

-Chetan K

You might be interested in
When multiplying a number by 10, move the decimal to the right. When multiplying a number by 0.1, move the decimal to the left.
ki77a [65]

Answer: 0.01 x 10 = .1

it moved ten spaces up or in simpler terms the decimal move one space to the right because the number is getting bigger.

Step-by-step explanation:

6 0
3 years ago
Read 2 more answers
Does the set of numbers form a Pythagorean triple? Explain.
Natali [406]

Answer:

NO

Step-by-step explanation:

Either 3, 5 and 6 satisfy the Pythagorean Theorem or they do not.  Here we start with a^2 + b^2 = c^2 and substitute 6 for c (because 6 is the largest of the set):  Is 3^2 + 5^2 = 6^2?  9 + 25 = 36?  NO

7 0
2 years ago
Triangle ABC is transformed to form triangle XYZ. If AC = BC and the perimeter of triangle ABC is 37 inches. What are the length
laiz [17]

Answer: xy=11 in.; yz= 13 in.; xz=13in.

Step-by-step explanation:

i’m pretty sure that’s it

8 0
3 years ago
9n - 8d<br> (n = 3 and d = 2
leva [86]

Answer:

11

Step-by-step explanation:

9(3)-8(2)

27-16

11

3 0
2 years ago
An unknown number y is 15 more than an unknown number x. The number y is also x less than 8. The equations to find x and y are s
melomori [17]
Add the equations to eliminate x
6 0
3 years ago
Read 2 more answers
Other questions:
  • Which situation best represents causation?
    7·2 answers
  • Given: Rectangle JKLM has an area of 36 square centimeters. Side is at least 4 centimeters long. Prove: KL 9 centimeters Assum
    7·1 answer
  • What is the missing angel in the triangle 37,90,
    14·1 answer
  • What was the price yesterday?
    6·1 answer
  • Martin deposits $200 in a savings account that earns 5% annual interest. Four years later, Cary deposits
    12·2 answers
  • -8 (1+11a)?????????????????
    6·2 answers
  • How many numbers are written from n to k, including n and k?
    5·1 answer
  • Could you explain to me how to do this ? I do not understand at all
    9·1 answer
  • Find all solutions of<br>- 2r +4-8=0​
    13·1 answer
  • Evaluate the expression when a=-15 and b=-5. b^2 / a + 5​
    5·1 answer
Add answer
Login
Not registered? Fast signup
Signup
Login Signup
Ask question!